Express Laplace Transform of y(t) in given form.

Click For Summary
The discussion focuses on solving the initial value problem (IVP) for the differential equation y''(t) + 2y'(t) + 10y(t) = r(t) with specified initial conditions and a piecewise function r(t). The Laplace transform of y(t) is sought in a specific form, and the user initially arrives at a similar expression but realizes a missing term due to a misapplication of the shifting theorem. The correct definition of r(t) is confirmed as t(H(t) - H(t-1)), leading to the conclusion that the Laplace transform should include an additional term. The user acknowledges the error and corrects the approach to include the necessary components in the transformation process.
chinye11
Messages
22
Reaction score
0

Homework Statement



y(t) solves the following IVP

y''(t) + 2y'(t) + 10y(t) = r(t)
y(0) = 2
y'(0) = 3
r(t) =
0 if t < 0
t if 0 ≤ t ≤ 1
0 if t > 1

Demonstrate that the laplace transform of y(t) is

Y(s) = \frac{2s+7}{s^{2}+2s+7} + \frac{e^{-s}}{s(s^{2}+2s+7)} + \frac{1}{s^{2}(s^{2}+2s+7)}+\frac{e^{-s}}{s^{2}(s^{2}+2s+7)}

Homework Equations



H(t) is the heaviside step function which is 0 when t is less than 0 and 1 when t is greater than 1, (undefined at 0.)

Tables of common laplace transforms are available on the internet e.g. http://www.rapidtables.com/math/calculus/laplace_transform.htm

3. Attempt at a Solution
Okay so I use the laplace transform on both sides of the equation with
r(t) defined as t H(t) - t H(t-1)

When I calculate this I finish with

\frac{2s+7}{s^{2}+2s+7} + \frac{1}{s^{2}(s^{2}+2s+7)}+\frac{e^{-s}}{s^{2}(s^{2}+2s+7)}

This is similar to the needed answer however I seem to have dropped a factor of se-s on the right side somewhere and I cannot see any mistakes in the algebra.

Is my definition of r(t) correct and am I correct to say that the laplace transform of t H(t) - t H(t-1) is

\frac{1}{s^{2}} -e^{s}(\frac{1}{s^{2}})?
 
Physics news on Phys.org
Yes, ##r(t) = t(H(t) - H(t-1))## but you are missing a term in the transform. You can check it yourself by just calculating the integral$$
\int_0^1 te^{-st}\, dt$$
 
OK, Thanks very much, I realized that I had applied the shifting theorem to an equation of the form:
y(t) (H(t-a)) when y(t-a) (H(t-a)) is required

This dropped a H(t-1) term which was the missing term.
 
Question: A clock's minute hand has length 4 and its hour hand has length 3. What is the distance between the tips at the moment when it is increasing most rapidly?(Putnam Exam Question) Answer: Making assumption that both the hands moves at constant angular velocities, the answer is ## \sqrt{7} .## But don't you think this assumption is somewhat doubtful and wrong?

Similar threads

  • · Replies 7 ·
Replies
7
Views
2K
  • · Replies 1 ·
Replies
1
Views
1K
  • · Replies 6 ·
Replies
6
Views
1K
Replies
2
Views
2K
  • · Replies 8 ·
Replies
8
Views
2K
  • · Replies 10 ·
Replies
10
Views
2K
  • · Replies 4 ·
Replies
4
Views
3K
  • · Replies 4 ·
Replies
4
Views
1K
  • · Replies 1 ·
Replies
1
Views
964
Replies
3
Views
2K